Number Theory 24

(IMO 2008 Shortlist)
[Bài toán]: Cho p\in \mathbb{P}, n\in \mathbb{Z^+},a,b,c\in \mathbb{Z} thỏa mãn a^n+pb=b^n+pc=c^n+pa.
CMR: a=b=c
                                                           Lời Giải
Giả sử a,b,c đôi một khác nhau.
Ta có: a^n+pb=b^n+pc=c^n+pa\Leftrightarrow \begin{cases}a^n-b^n=p(c-b)\\b^n-c^n=p(a-c)\\c^n-a^n=p(b-c)\end{cases}
\Rightarrow -p^3=\displaystyle{\frac{a^n-b^n}{a-b}.\frac{b^n-c^n}{b-c}.\frac{c^n-a^n}{c-a}}\qquad (1)
Xét n lẻ \Rightarrow a^n-b^n,a-b cùng dấu \Rightarrow VP_{(1)}>0VT_{(1)}<0 (vô lí)
Xét n chẵn:
\blacktriangleright Xét p lẻ, p\geq 3. Ta có: -p^3=\prod (a^{n-1}+a^{n-2}b+...+ab^{n-2}+b^{n-1})
Ta thấy a^{n-1}+a^{n-2}b+...+ab^{n-2}+b^{n-1} (2) lẻ mà số các số trong tổng (2) chẵn do n chẵn \Rightarrow a,b khác dấu \Rightarrow a-b\equiv 1\pmod 2
Tương tự \Rightarrow 3\equiv (a-b)+(b-c)+(c-a)\equiv 0\pmod 2 (vô lí)
\blacktriangleright Xét p chẵn \Rightarrow p=2. Đặt n=2k với k\in \mathbb{Z^+},n\geq 2.
(*) Nếu trong ba số có một số bằng 0. Giả sử a=0.
Ta có: \begin{cases}b^n+2c=c^n\\2b=c^n\end{cases}\Rightarrow c^{n^2}+2^{n+1}c=2^nc^n\Leftrightarrow (c^{n-1})^{n+1}-4+4=2^n(c^{n-1}-2)
\Leftrightarrow (c^{n-1}-2)A+4=2^n(c^{n-1}-2)\Leftrightarrow (c^{n-1}-2)(2^n-A)=4 (dễ dàng thấy vô lí)
(*) Nếu cả ba số đều khác 0\Rightarrow \left | a \right |,\left | b \right |,\left | c \right |\geq 1
+, Với k\geq 2. Ta có: \displaystyle{8=\left | \prod \frac{a^n-b^n}{a-b} \right |=\left | \prod (a+b)\frac{a^{2k}-b^{2k}}{a^2-b^2} \right |}
=\left | (a+b)(b+c)(c+a) \right |\left | \prod (a^{2k-2}+...+b^{2k-2}) \right |\geq 8\left | (a+b)(b+c)(c+a) \right |
\Leftrightarrow \left | (a+b)(b+c)(c+a) \right |\leq 1\rightarrow \text{False}
+, Với k=1\Rightarrow (a+b)(b+c)(c+a)=-8. Từ giả thiết, dễ dàng thấy a,b,c cùng tính chẵn lẻ \Rightarrow \left | a+b \right |=\left | b+c \right |=\left | c+a \right |=2\Rightarrow a=b=c.
Vậy ta có Q.E.D \blacksquare

Number Theory 22

(USA Team Selected Test 2003)
[Bài toán]: Tìm bộ ba số nguyên tố p,q,r thỏa mãn điều kiện: p\mid q^r+1,q\mid r^p+1,r\mid p^q+1
                                                                     Lời Giải
\blacktriangleright Bổ đề: Cho a,b,c\in \mathbb{P},c lẻ sao cho c\mid a^b+1 thì 2b\mid c-1;c\mid a^2-1
\rightarrow CM: Đặt d=\text{ord}_c(a)\qquad (d\in \mathbb{Z^+})
Ta có: c\mid a^b+1;c>2\Rightarrow a^b\equiv -1\pmod c,a^{2b}\equiv 1\pmod c
\Rightarrow d\mid 2b;d\mid b\Rightarrow d\in \{2;2b\}
+, Với d=2bd\mid c-1 (vì \text{ord}_c(a)\mid \phi(c)=c-1 do c\in \mathbb{P}) \Rightarrow 2b\mid c-1
+, Với d=2\Rightarrow a^2\equiv 1\pmod c\Rightarrow c\mid a^2-1
\blacktriangleright Áp dụng:
(*) Nếu có ít nhất 2 số bằng nhau, dễ dàng ta thấy điều vô lí!
(*) Không mất tính tổng quát, giả sử r<p<q
+, Nếu cả p,q,r đều lẻ. Ta có: q\mid r^p+1 \Rightarrow \begin{cases} 2p\mid q-1\\q\mid r^2-1\end{cases} \Rightarrow \begin{cases} q-1\geq 2p\\q\mid (r-1)(r+1)\end{cases}
Dễ dàng thấy trường hợp thứ nhất vô lí vì q<p
Ở trường hợp thứ hai, ta thấy \gcd(r-1;r+1)=2,p\in \mathbb{P}\Rightarrow q\mid \displaystyle{\frac{r-1}{2}} hoặc q\mid \displaystyle{\frac{r+1}{2}}q<\displaystyle{\frac{r-1}{2}<\frac{r+1}{2}}<r (trái với giả thiết nên trường hợp này loại)
+, Nếu trong 3 số có 1 số chẵn mà r<q<p\Rightarrow r=2
Ta có: r\mid p^q+1\Rightarrow \begin{cases}4=2q\mid r-1\\r\mid p^2-1\end{cases} p\mid q^r+1\Rightarrow \begin{cases} 2r\mid p-1\\p\mid q^2-1\end{cases} \Leftrightarrow p\mid 3\Rightarrow p=3
\Rightarrow r\mid 10p\in \mathbb{P}\Rightarrow r\in \{2;5\}r>2\Rightarrow r=5 (Thử lại thỏa mãn)
Vậy (p,q,r)=(2;3;5) và hoán vị thỏa mãn ycđb \blacksquare

 

Number Theory 15

(Olympic Chuyên KHTN 2014)
Tìm tất cả bộ 3 số (x,n,p) với x,n\in \mathbb{Z^+},p\in \mathbb{P} thỏa mãn:
x^3+2x=3(p^n-1)
                                                             Lời Giải
Ta có: x^3+2x=3(p^n-1) \Leftrightarrow (x+1)(x^2-x+3)=3p^n
Đặt \gcd(x+1;x^2-x+3)=d(d\in \mathbb{N^*})
\Rightarrow d\mid x+1;d\mid x^2-x+3=(x+1)(x-2)+5 d\mid 5 d\in \begin{Bmatrix} 1;5 \end{Bmatrix}
(*) Xét d=1
+, Xét p=3\Rightarrow (x+1)(x^2-x+3)=3^{n+1}=1.3^{n+1}x\in \mathbb{Z^+}\Rightarrow x+1>1, x^2-x+3>1 \Rightarrow Trường hợp này không xảy ra
+, Xét p\neq 3
\begin{cases} x+1=3 \\ x^2-x+3=p^n \end{cases} \Leftrightarrow \begin{cases}n=1 \\ x=2 \\ p=5 \end{cases}
\begin{cases} x+1=p^n \\ x^2-x+3=3 \end{cases} \Leftrightarrow \begin{cases}x=1 \\ p=2 \\ n=1 \end{cases}
(*) Xét d=5\Rightarrow 3p^n\vdots 5 \Rightarrow p=5, với n\geq 2
Ta có: (x+1)(x^2-x+3)=3.5^n=3.5.5^{n-1} nên xét các trường hợp sau:
\begin{cases} x+1=5 \\ x^2-x+3=3.5^{n-1}\end{cases} \Leftrightarrow \begin{cases} x=4\\ n=2\\ p=5\end{cases}
• \begin{cases} x+1=3.5^{n-1} \\ x^2-x+2=5\end{cases} \Leftrightarrow \begin{cases} x=2\\ n=1\\ p=5\end{cases}
• \begin{cases} x+1=3.5 \\ x^2-x+3=5^{n-1}\end{cases} \Leftrightarrow \begin{cases} x=14\\ 5^{n-1}=185\end{cases} (vô lí)
• \begin{cases} x+1=5^{n-1} \\ x^2-x+3=15\end{cases} \Leftrightarrow \begin{cases} x=4\\ n=2\\ p=5\end{cases}
Vậy phương trình có nghiệm \boxed{(x,n,p)=\{ (4;2;5); (1;1;2); (2;1;5)\}} \blacksquare

Number Theory 13

(Balkan MO 2005)
[Bài toán]: Tìm p nguyên tố sao cho p^2-p+1 là lập phương một số tự nhiên.
                                                          Lời Giải
Đặt p^2-p+1=a^3(a\in \mathbb{N}) \qquad (1)
-Với a=0 không thỏa mãn
(1) \Leftrightarrow p(p-1)=(a-1)(a^2+a+1)\qquad (2)
\Rightarrow p\mid (a-1)(a^2+a+1) \Rightarrow 2 trường hợp:
\blacktriangleright Nếu p\mid a-1. Đặt a-1=pk(k\in \mathbb{N})
Ta có: (1)\Leftrightarrow p^2-p+1=(pk+1)^3=p^3k^3+3p^2k^2+3pk+1
\Leftrightarrow p-1=p^2k^3+3pk^2+3k
+, Nếu k\geq 2 \Rightarrow p-1\geq 8p^2+12p+6\Leftrightarrow 8p^2+11p+7\leq 0 (vô lí)
+, Nếu k=1 \Rightarrow p-1=p^2+3p+3 \Leftrightarrow p^2+2p+4=0 (vô lí)
+, Nếu k=0 \Rightarrow p=1 (vô lí)
Vậy trường hợp này không xảy ra.
\blacktriangleright Nếu p\mid a^2+a+1 \qquad (3)
Từ (2) \Rightarrow a-1\mid p(p-1)\gcd(p;p-1)=1 nên ta có 2 trường hợp:
+, Xét a-1\mid p \Rightarrow p\geq a-1 \Rightarrow p^2-p+1\leq (p+1)^3
\Leftrightarrow p(p^2+2p+4)\leq 0 (vô lí)
+, Xét a-1\mid p-1. Đặt p-1=q(a-1)(q\in \mathbb{Z^+}) \Leftrightarrow p=q(a-1)+1\qquad (4)
Từ (3),(4) \Rightarrow \displaystyle{\frac{a^2+a+1}{aq-q+1}}\in \mathbb{Z^+}
\Rightarrow \displaystyle{\frac{a^2+aq+q}{aq-q+1}=\frac{a(aq-q+1)+2(aq-q+1)+3q-a-2}{aq-q+1}}
=a+2+\displaystyle{\frac{3q-a-2}{aq-q+1}}\in \mathbb{Z^+}
\Rightarrow aq-q+1\mid 3q-a-2
\Rightarrow \left | 3q-a-2 \right |\geq aq-q+1
• Nếu 3q-a-2\geq aq-q+1 \Leftrightarrow q(4-a)\geq 3+a \Leftrightarrow (q+1)(a-4)\leq -7
Thay a=1;2;3 đều không thỏa mãn \Rightarrow a\geq 4 \Rightarrow (q+1)(a-4)\geq 0 (vì q\in \mathbb{Z^+}). Ta có điều mâu thuẫn nên trường hợp này không xảy ra.
• Nếu 2+a-3q\geq aq-q+1 \Leftrightarrow a+1\geq q(a+2) (vô lí vì a+1<a+2,q\in \mathbb{Z^+})
• Nếu 2+a-3q=0 \Leftrightarrow 2+a=3q
Ta có: (4)\Leftrightarrow 3p=3q(a-1)+3=(a-1)(2+a)+3=a^2+a+1
\Rightarrow (2)\Leftrightarrow p(p-1)=3p(a-1) \Leftrightarrow p=3a-2
\Rightarrow 3(3a-2)=a^2+a+1\Leftrightarrow (a-1)(a-7)=0\Leftrightarrow a=1;7
\Rightarrow a=7\Rightarrow p=19 (Thay vào (1))
Kết luận: Vậy \boxed {p=19} thỏa mãn \blacksquare

 

Number Theory 6

(Tuyển sinh Chuyên Thái Bình 2014-2015)

[Bài toán]: Cho a,b,c,d nguyên dương thỏa mãn: a^2+ab+b^2=c^2+cd+d^2.
CMR: a+b+c+d là hợp số.
                                                             Lời giải
Ta có: a^2+ab+b^2=c^2+cd+d^2
\Leftrightarrow a^2+2ab+b^2-ab=c^2+2cd+d^2-cd
\Leftrightarrow (a+b)^2-(c+d)^2=ab-cd
\Leftrightarrow (a+b+c+d)(a+b-c-d)=ab-cd

a,b,c,d\in \mathbb{Z^+}\Rightarrow a+b-c-d=\displaystyle{\frac{ab-cd}{a+b+c+d}}
a+b-c-d\in \mathbb{Z}\Rightarrow \displaystyle{\frac{ab-cd}{a+b+c+d}}\in \mathbb{Z}

Giả sử a+b+c+d là số nguyên tố \Rightarrow a+b+c+d|ab-cd
Từ gt\Leftrightarrow 3(ab-cd)=c^2+d^2-2cd-a^2-b^2+2ab
\Leftrightarrow 3(ab-cd)=(c-d)^2-(a-b)^2=(c-d-a+b)(c-d+a-b)
Từ hai điều trên \Rightarrow a+b+c+d|(c-d-a+b)(c-d+a-b)

a+b+c+d là số nguyên tố nên ta có 2 trường hợp sau:

+, Ta có:
a+b+c+d|c-d+a-b \Leftrightarrow (a+b+c+d)|2(a+c)
2<a+b+c+d,a+c<a+b+c+d nên trường hợp này vô lí.

+, Ta có: c-d-a+b\vdots a+b+c+d. Tương tự, ta có điều vô lí.

Kết luận: Vậy giả sử sai, ta có a+b+c+d là hợp số (Q.E.D) \blacksquare.

 

Number Theory 5

(Tuyển sinh Chuyên Phổ thông Năng khiếu 2014-2015)
Cho các số nguyên dương a,b,c thỏa mãn $latex \frac{1}{a}+\frac{1}{b}=\frac{1}{c}$ (1)
a. CMR: a+b không thể là số nguyên tố.
b. CMR: Nếu c>1 thì a+c,b+c không thể đồng thời là số nguyên tố.
                                                              Lời giải
a. Ta có: \frac{1}{a}+\frac{1}{b}=\frac{1}{c}(1)\Leftrightarrow c(a+b)=ab(2)
Giả sử a+b là số nguyên tố.
Ta có: (1)\rightarrow \frac{1}{a}< \frac{1}{c}\Leftrightarrow a>c;\frac{1}{b}<\frac{1}{c}\Leftrightarrow b>c
\Rightarrow a+b>ca+b\in \mathbb{P}\Rightarrow \text {gcd}(a+b;c)=1
Từ (2)\Rightarrow \left\{\begin{matrix} c(a+b)\vdots a & \\ c(a+b)\vdots b & \end{matrix}\right.
\Rightarrow \left\{\begin{matrix} a\vdots b & \\ b\vdots a & \end{matrix}\right.
\Rightarrow a=b\Rightarrow a+b=2a\vdots 2\Rightarrow a+b không là số nguyên tố.
Vậy giả sử sai, ta có Q.E.D

b. Ta có: c(a+b)=ab\Leftrightarrow c^2+2ab=2c(a+b)+c^2
\Leftrightarrow a(b+c)+c(b+c)=2ab+c^2
\Leftrightarrow (a+c)(b+c)=2c(a+b)+c^2=c(2a+2b+c)
\Rightarrow (a+c)(b+c)\vdots c (3)
Giả sử a+c;b+c đồng thời là số nguyên tố \Rightarrow \text {gcd}(a+c;b+c)=1 (4)
Từ (3),(4) \Rightarrow \left\{\begin{matrix} a+c\vdots c & \\ b+c\vdots c & \end{matrix}\right.
c>1\Rightarrow a+c hoặc b+c là hợp số.
Vậy giả sử sai, ta có Q.E.D

 

 

 

Number Theory 2

(THTT 434 tháng 8-2013)
Cho biết p nguyên tố. Tìm x,y nguyên sao cho x^2(x^2+y^2)=y^{p+1}. (1)
Lời giải

Đặt x^2=t (t\geq 0)

(1) \Leftrightarrow t^2+ty^2-y^{p+1}=0.
Coi phương trình trên bậc hai ẩn t.
Ta có: \Delta=y^4+4y^{p+1}
Để phương trình (1) có nghiệm nguyên thì \Delta phải là số chính phương.
Đặt y^4+4y^{p+1}=a^2(a\in \mathbb{N}) (2)

* Nếu p=2\Rightarrow  (1) \Leftrightarrow y^4+4y^3=a^2\Leftrightarrow y^2(y^2+4y)=a^2
\Rightarrow y^2+4y là số chính phương.
Đặt y^2+4y=b^2(b\in \mathbb{N})
Ta có: (y+2)^2-4=b^2\Leftrightarrow (y+2-b)(y+2+b)=4
Vì y nguyên, b là số tự nhiên \Rightarrow y+2-b\leq y+2+b
y+2-b+y+2+b=2(y+2) chẵn \Rightarrow 2+y-b; 2+y+b cùng tính chẵn lẻ.
Xét các trường hợp:
+, \left\{\begin{matrix}  y+2-b=-2 & \\  y+2+b=-2 &  \end{matrix}\right.\Leftrightarrow y=4\Rightarrow (1)\Leftrightarrow x^4+16x^2+64=0\Leftrightarrow (x^2+8)^2=0\Leftrightarrow x^2=-8 (vô lí)
+, \left\{\begin{matrix}  y+2-b=2& \\  y+2+b=2 &  \end{matrix}\right.\Leftrightarrow y=0\Rightarrow x=0

*Nếu p>2\Rightarrow p lẻ. Đặt p=2k+1(k\in \mathbb{Z^+})
Ta có: \Delta=y^4+4^{2k+2}=y^4(1+4y^{2k-2})=a^2
\Rightarrow 1+4y^{2k-2} là số chính phương.
Đặt 1+4y^{2k-2}=c^2(c\in \mathbb{N}) (3)
Ta có: (3) \Leftrightarrow (c-2y^{k-1})(c+2y^{k-1})=1
Lập luận tương tự như trường hợp trên, ta có các trường hợp sau:
$latex \left\{\begin{matrix} c-2y^{k-1}=\pm 1 & \\ c+2y^{k-1}=\pm 1 & \end{matrix}\right.\Leftrightarrow 4y^{k-1}=0\Leftrightarrow y=0\Leftrightarrow x=0$
Vậy phương trình có nghiệm là (x,y)=(0;0)

Number Theory 1

Cho n là số tự nhiên lẻ và n có thể biểu diễn không ít hơn 2 cách là tổng của hai số chính phương.
CMR: n là hợp số.
                                                                                                                  Lời giải
Giả sử
n là số nguyên tố.

Vì n có thể biểu diễn không ít hơn 2 cách là tổng của 2 số chính phương nên ta đặt n=a^2+b^2=c^2+d^2a,b,c,d\in \mathbb{Z}
\Rightarrow n^2=(a^2+b^2)(c^2+d^2)=a^2c^2+a^2d^2+b^2c^2+b^2d^2=(ac+bd)^2+(ad-bc)^2\Rightarrow n^2\geq (ac+bd)^2 (1)

Ta có: (ac+bd)(ad+bc)=ab(c^2+d^2)+cd(a^2+b^2)=(ab+cd)n
\Rightarrow \begin{bmatrix} ac+bd\vdots n & & \\ ad+bc\vdots n & & \end{bmatrix}

Không mất tính tổng quát, giả sử  ac+bd\vdots n\Rightarrow (ac+bd)^2\geq n^2 (2)

Từ (1) và (2) \Rightarrow (ac+bd)^2=n^2
\Leftrightarrow ad=bc\Leftrightarrow \frac{a}{c}=\frac{b}{d}

+, Nếu a>c\Rightarrow b>d\Rightarrow a^2+b^2> c^2+d^2 (mâu thuẫn)

+, Nếu a<c, tương tự, cũng có điều mâu thuẫn!

Vậy giả sử sai, có dpcm